Difference between revisions of "2004 AMC 10B Problems/Problem 6"

(Solution)
m (Solution)
Line 9: Line 9:
 
Using the fact that <math>n! = n\cdot (n-1)!</math>, we can write:
 
Using the fact that <math>n! = n\cdot (n-1)!</math>, we can write:
  
* <math>A=98! \cdot (99\cdot 98!) = 99 \cdot (98!)^2 = 11\cdot(3\cdot98!)^2</math>
+
<cmath>\begin{aligned} A&=98! \cdot (99\cdot 98!) = 99 \cdot (98!)^2 = 11\cdot3^2\cdot(98!)^2 \\ B&=100 \cdot 99 \cdot (98!)^2 = 11\cdot10^2\cdot3^2\cdot( 98!)^2 \\ C&=100\cdot (99!)^2 = 10^2\cdot (99!)^2\\ D&=101\cdot 100\cdot (99!)^2 = 101 \cdot 10^2 \cdot (99!)^2\\ E& =101\cdot (100!)^2 \end{aligned}</cmath>
  
* <math>B=100 \cdot 99 \cdot (98!)^2 = 11\cdot (30\cdot 98!)^2 </math>
+
We see that <math>\boxed{\mathrm{(C) \ } 99! \cdot 100!}</math> is a square, and because <math>11</math>, and <math>101</math> are primes, none of the other four choices are squares.
 
 
* <math>C=100\cdot (99!)^2 = (10\cdot 99!)^2</math>
 
 
 
* <math>D=101\cdot 100\cdot (99!)^2 = 101 \cdot(10\cdot 99!)^2</math>
 
 
 
* <math>E=101\cdot (100!)^2</math>
 
 
 
Clearly <math>\boxed{\mathrm{(C) \ } 99! \cdot 100!}</math> is a square, and as <math>11</math>, and <math>101</math> are primes, none of the other four are squares.
 
  
 
== See also ==
 
== See also ==

Revision as of 13:49, 20 January 2014

Problem

Which of the following numbers is a perfect square?

$\mathrm{(A) \ } 98! \cdot 99! \qquad \mathrm{(B) \ } 98! \cdot 100! \qquad \mathrm{(C) \ } 99! \cdot 100! \qquad \mathrm{(D) \ } 99! \cdot 101! \qquad \mathrm{(E) \ } 100! \cdot 101!$

Solution

Using the fact that $n! = n\cdot (n-1)!$, we can write:

\begin{aligned} A&=98! \cdot (99\cdot 98!) = 99 \cdot (98!)^2 = 11\cdot3^2\cdot(98!)^2 \\ B&=100 \cdot 99 \cdot (98!)^2 = 11\cdot10^2\cdot3^2\cdot( 98!)^2 \\ C&=100\cdot (99!)^2 = 10^2\cdot (99!)^2\\ D&=101\cdot 100\cdot (99!)^2 = 101 \cdot 10^2 \cdot (99!)^2\\ E& =101\cdot (100!)^2 \end{aligned} (Error compiling LaTeX. Unknown error_msg)

We see that $\boxed{\mathrm{(C) \ } 99! \cdot 100!}$ is a square, and because $11$, and $101$ are primes, none of the other four choices are squares.

See also

2004 AMC 10B (ProblemsAnswer KeyResources)
Preceded by
Problem 5
Followed by
Problem 7
1 2 3 4 5 6 7 8 9 10 11 12 13 14 15 16 17 18 19 20 21 22 23 24 25
All AMC 10 Problems and Solutions

The problems on this page are copyrighted by the Mathematical Association of America's American Mathematics Competitions. AMC logo.png